LSAT and Law School Admissions Forum

Get expert LSAT preparation and law school admissions advice from PowerScore Test Preparation.

 Administrator
PowerScore Staff
  • PowerScore Staff
  • Posts: 8915
  • Joined: Feb 02, 2011
|
#23606
Complete Question Explanation

Justify the Conclusion-PR. The correct answer choice is (E)


The archaeologist grants that the restoration project the corporation offers to fund would conform to current theories about the site, but on the basis that many parts of the site contain unexamined information, the archaeologist concludes that the offer should be rejected.

The archaeologist is probably concerned that the new evidence could have a significant impact on the theories about how the site appeared. The archaeologist seems to operate on the broad assumption that one should not reconstruct archaeological sites until as much evidence as possible is evaluated. Since you are asked to find the principle that would justify the argument, you should locate a choice that reinforces the archaeologist's broad assumption.

Answer choice (A): The archaeologist does not seem to have a problem with commercial interests; the archaeologist is merely concerned with accuracy. Furthermore, even if the stimulus clearly indicated (it does not) that the corporation would own the site, this choice does not respond to the argument, which is based on the idea of evidence, not ownership.

Answer choice (B): This response reinforces the idea that restorations should represent the most ancient period of a site's history, but the stimulus promotes the idea that the restorations should represent the site at the relevant civilization's height.

Answer choice (C): This response suggests that the corporation is wrong to make a judgment concerning the height of the civilization in question, but the archaeologist actually seems to accept that judgment and only argues that more evidence should be evaluated, presumably to paint a better picture of that civilization at its height.

Answer choice (D): Since the archaeologist's argument was concerned with the state of evidence rather than the source of funding, the idea that the corporation should not be the source of funding is irrelevant to the argument, and this choice is wrong even if you incorrectly presume it supports the archaeologist's conclusion. In fact, this choice does not even support the conclusion, because it is entirely possible that the corporation in question actually has a true concern for the site's history.

Answer choice (E): This is the correct answer choice. The archaeologist is concerned with the potential of new evidence, and the principle that the risk of losing evidence for future theories outweighs the advantages of displaying the results of theories based on current evidence speaks to that issue. You cannot plausibly deny that restoration and public access incurs some risk to evidence.
 reop6780
  • Posts: 265
  • Joined: Jul 27, 2013
|
#15776
The correct answer is E while I chose C.

I kept both of them as contenders without 100 percent certainty.

I excluded E because I did not think "the risk of losing evidence" is not quite the same as "unexamined evidence" in the stimuli.

How is it certain that restoration may lead to the risk of losing evidence while the stimuli merely states that there are some unexamined evidence?

In other words, the principle of E cannot be applied unless restoration of a building contaning unexmained evidence is risking of losing evidence.

Also, should I attack the direct premise of the conclusion only for the justification?

Is answer C not touching the premise of the stimuli at all?
 Steve Stein
PowerScore Staff
  • PowerScore Staff
  • Posts: 1153
  • Joined: Apr 11, 2011
|
#15782
Hi,

In that one, you're looking for a principle to justify the argument that the restoration should not take place, based on the fact that many parts of the site have yet to be examined. The correct answer provides that the risk of losing that evidence would not be justified by the restoration, justifying the archaeologist's argument. Answer choice (C), on the other hand, uses very strong language (no one should make such judgments) and doesn't support the author's argument, that the restoration should not take place.

I hope that's helpful! Please let me know whether this is clear—thanks!

~Steve
 reop6780
  • Posts: 265
  • Joined: Jul 27, 2013
|
#15831
Yea, it was helpful! :) Thank you, Steve.
 Steve Stein
PowerScore Staff
  • PowerScore Staff
  • Posts: 1153
  • Joined: Apr 11, 2011
|
#16139
Hi,

Thanks for your message--I appreciate your response and I'm glad that was helpful!

Steve
 Etsevdos
  • Posts: 62
  • Joined: Oct 22, 2017
|
#41476
Back to C. If you cannot make a judgement about what constituted the height of a civilization, you are unable to do the restoration discussed in premise?

E: Aren't we assuming there is a risk in stimulus? Maybe as they build the replicas, they will find this evidence and preserve it better?
 nicholaspavic
PowerScore Staff
  • PowerScore Staff
  • Posts: 271
  • Joined: Jun 12, 2017
|
#42096
Hi etsevdos,

I don't see any language to support the argument that risk is assumed in the stimulus. There is a statement about the height of civilization in the stim's sentence that you need to accept as true. Challenging that statement would not be one of your goals for a Justify question of this type. Thanks for the great question and I hope this helps.
 Claire Horan
PowerScore Staff
  • PowerScore Staff
  • Posts: 408
  • Joined: Apr 18, 2016
|
#42101
Hi Etsevdos,

I just want to add something to Nicolas's excellent response because I understand your question to be "doesn't E assume there is a risk even though risk is not mentioned in the stimulus?"

Here's why risk does not actually have to be mentioned. The archaeologist offers an option, accepting funding (1st sentence). The archaeologist then gives one pro of accepting that funding (2nd sentence). The archaeologist then concludes that, despite this pro, they should go with another option, rejecting the funding (1st half of 3rd sentence). A con of accepting the funding is stated: "many parts of the site contain unexamined evidence." The question stem asks us to justify the conclusion—in other words, we need to fill in a gap so that the conclusion makes logical sense. The conclusion will make logical sense if the con outweighs the pro.

We need an answer choice that explains why "many parts of the site contain unexamined evidence" is a con. If (E) is valid (and the question stem asks us to assume that it each answer choice is valid) there is a risk that development will cause unexamined evidence to be lost.

Get the most out of your LSAT Prep Plus subscription.

Analyze and track your performance with our Testing and Analytics Package.